LSAT and Law School Admissions Forum

Get expert LSAT preparation and law school admissions advice from PowerScore Test Preparation.

User avatar
 Dave Killoran
PowerScore Staff
  • PowerScore Staff
  • Posts: 5852
  • Joined: Mar 25, 2011
|
#88739
Complete Question Explanation
(The complete setup for this game can be found here: lsat/viewtopic.php?f=232&p=88732)

The correct answer choice is (E)

If exactly three students review U, those students must be L, M, and O. Here’s why: K cannot review any additional plays, and if J reviews U, then neither L nor M could review U, leading to an outcome where only two students reviewed U. Thus, the scenario in this question is as follows:

G4-Q21-d1.png

Answer choices (A), (B), and (C) are eliminated based on the analysis above. Answer choice (D) is also eliminated, because L already reviews U, and L can review only 1 play.

Hence, answer choice (E) is proven correct by process of elimination.
You do not have the required permissions to view the files attached to this post.
 cfu1
  • Posts: 15
  • Joined: Mar 28, 2017
|
#34320
I'm having a little trouble figuring out the solution for question #21 (I guessed and somehow got it right).

From my original diagram, I have that Jiang, Kramer, and Lopez all review exactly one play. Megregian reviews two plays, and the number of plays reviewed by O'Neill is ambiguous. With that being said, if exactly three students review Undulation, then wouldn't Lopez, Megregian, and O'Neill be the ones reviewing Undulation?

If that holds true, then O'Neill reviews at least Undulation and Tamerlane. How can O'Neill also review Sunset? If he does, then the condition stating that "exactly two of the students review exactly the same play or plays as each other" wouldn't be met?

Here is my diagram for this problem:

J - S
K - T
L - U
M - U, T
O - U, T

Please let me know what I'm missing! Thanks so much.
User avatar
 Jonathan Evans
PowerScore Staff
  • PowerScore Staff
  • Posts: 726
  • Joined: Jun 09, 2016
|
#34382
Hi, CFU,

Good question!

You've done an excellent job at this game. You're just missing one possibility:
  • J = {T}, K = {T}, L = {U}, M = {S, U}, O = {S, T, U}
Does this make sense? I hope this helps!
 cfu1
  • Posts: 15
  • Joined: Mar 28, 2017
|
#34383
Oh ah well I was dumb, thanks so much for your help, Jonathan!
 elyssa1
  • Posts: 6
  • Joined: Dec 08, 2017
|
#45280
Hi- I'm trying to figure out what I didn't get on this problem. (I discarded the answer sheet... it's E right? If not, then maybe I need to start over.) Reviewing Jonathan's possibility above, I don't understand how that scenario can work with the last rule, and maybe I'm interpreting the rule incorrectly. With this rule along with that scenario, we have M & O being the two students that review the exact same plays as one another? I think I'm confused because O also reviews Tamerlane, and in understanding this rule, I thought that it meant all and only - so the two students had to see the exact same films and no other ones (hence not allowing O'Neill to review Tamerlane). Is it just that I'm interpreting the rule incorrectly? Hope I didn't make this too logically complicated, thank you!
 Francis O'Rourke
PowerScore Staff
  • PowerScore Staff
  • Posts: 471
  • Joined: Mar 10, 2017
|
#45283
Hi Elyssa,

I think you are understanding this rule just fine! It may be that you are not considering the entire game board.

The rule told us that we needed one pair of students who have identical play assignments. The rule did not specify which students that pair needed to be however.

cfu1 gave us a possible assignment in which M and O review the same plays as each other. Jonathon gave us a possible assignment in which J and K review the same plays as each other. The rule still holds in this example since Jiang and Kramer both review only Tamerlane.

Let me know if this helps! :-D

Get the most out of your LSAT Prep Plus subscription.

Analyze and track your performance with our Testing and Analytics Package.